Finding a basis in ImT using Gaussian Elimination

Click For Summary
The discussion centers on finding the image of a transformation (ImT) basis using Gaussian elimination on a given matrix. The user successfully reduced the matrix to row-echelon form, identifying two basis vectors: [-1, 2, 0] and [3, 0, -6]. They express confusion over why they cannot further simplify the matrix to obtain an identity form, which would suggest three basis vectors instead of the correct two. Clarification is sought on the definition of "ImT basis," with the understanding that it refers to the basis that spans the image of the transformation defined by the matrix. The conversation highlights the importance of recognizing pivot positions in determining the basis of the transformation.
JoshMaths
Messages
26
Reaction score
0

Homework Statement



$$
\begin{pmatrix}
-1&3&0\\
2&0&-1\\
0&-6&1
\end{pmatrix}
$$

Finding the ImT basis of this

The Attempt at a Solution



I got it down to

$$
\begin{pmatrix}
1&0&-1/2\\
0&1&1/6\\
0&0&1
\end{pmatrix}
$$

I know that by the principle of having pivots as the only non-zero entities in their respective columns this makes that column one of the basis vectors. So answer is

[-1,2,0] [3,0,-6]

What i don't understand is why (In the r-echelon form) i cannot subtract -1/2(Row 3) from Row 1 and Subtract 1/6(Row 3) from Row 2 to give the Guass-Jordan form or Identity form which would imply that the entire first matrix was a basis for itself right? Meaning the basis contains three vectors instead of the actual two is contains in the correct answer.

Thanks, and i hope the question has come out clearly, just say if clarification is needed.

Josh
 
Last edited:
Physics news on Phys.org
What is 'ImT basis'?
The image of some basis under T?
 
No it's just the basis over T i guess. The generic basis that spans T(v) where v is an arbitrary vector and the matrix for T is the above.
 
Question: A clock's minute hand has length 4 and its hour hand has length 3. What is the distance between the tips at the moment when it is increasing most rapidly?(Putnam Exam Question) Answer: Making assumption that both the hands moves at constant angular velocities, the answer is ## \sqrt{7} .## But don't you think this assumption is somewhat doubtful and wrong?

Similar threads

  • · Replies 7 ·
Replies
7
Views
1K
  • · Replies 4 ·
Replies
4
Views
2K
  • · Replies 2 ·
Replies
2
Views
1K
  • · Replies 37 ·
2
Replies
37
Views
6K
  • · Replies 3 ·
Replies
3
Views
2K
  • · Replies 3 ·
Replies
3
Views
2K
Replies
4
Views
2K
Replies
8
Views
2K
  • · Replies 3 ·
Replies
3
Views
2K
  • · Replies 12 ·
Replies
12
Views
3K